ANTR510 E3 Qbank

Réussis tes devoirs et examens dès maintenant avec Quizwiz!

A the infraspinatus laterally rotates the glenohumeral j.

A 45-year-old male presents to his physician complaining of right shoulder pain that began after chopping wood 2 days prior. Examination of the right upper extremity shows no obvious bone deformities or point tenderness. The pain is reproduced when the patient is asked to externally rotate the shoulder against resistance; there is no weakness. Injury to which of the following muscles is the most likely cause of his symptoms? A. infraspinatus B. pectoralis major C. subscapularis D. supraspinatus E. trapezius

D The radial nerve innervates the extensors of the digits, which are located in the posterior compartment of the forearm

An Emergency Department physician examining a patient with a bullet wound in the arm, asks the patient to extend the index and pinky fingers with the ring and middle fingers flexed. The patient is able to attain this position. This shows that which of the following nerves is intact? A Axillary B Medial C Musculocutaneous D Radial E Ulnar

B

Correctly trace a drop of blood as it flows through the following vessels:1: Axillary artery2: Brachial artery3: Digital arteries4: Subclavian artery5: Superficial palmar arch6: Radial artery A 4, 2, 1, 5, 6, 3 B 4, 1, 2, 6, 5, 3 C 1, 4, 2, 6, 3, 5 D 4, 1, 2, 6, 3, 5 E 1, 2, 4, 5, 6, 3

B The cephalic vein passes within the detlopectoral triangle to join the proximal portion of the axillary vein.

The cephalic vein is a tributary to which of the following veins? A brachial B axillary C subclavian D lateral brachial

D,E

Which of the following muscles are responsible for pronation? (select all that apply) A biceps brachii B brachialis C brachioradialis D pronator teres E pronator quadratus F supinator G triceps brachii

A,F

Which of the following muscles are responsible for supination? (select all that apply) A biceps brachii B brachialis C brachioradialis D pronator teres E pronator quadratus F supinator G triceps brachii

C Serratus anterior protracts the scapula and is often well developed in boxers.

Which of the following muscles protracts the scapula (as during punching/jabbing)? A rhomboids B trapezius C serratus anterior D deltoid

E,F,I

Which of the following muscles are responsible for adduction at the glenohumeral joint? (select all that apply) A biceps brachii long head B coracobrachialis C deltoid D infraspinatus E latissimus dorsi F pectoralis major G subscapularis H supraspinatus I teres major J teres minor

C,E,H,I

Which of the following muscles are responsible for extension at the glenohumeral joint? (select all that apply) A biceps brachii long head B coracobrachialis C deltoid D infraspinatus E latissimus dorsi F pectoralis major G subscapularis H triceps brachii I teres major J teres minor

C A shoulder separation occurs at the acromioclavicular joint. A shoulder dislocation occurs at the glenohumeral joint.

The star football quarterback is tackled and driven to the ground by the defensive player, who fell on top of the quarterback. The quarterback's right shoulder is the first part of his body to hit the ground. When the quarterback walks off the field, his right arm hangs limply by his side. The team trainer diagnoses the injury as a severe shoulder separation. A shoulder separation occurs at which of the following joints? A glenohumeral B sternoclavicular C acromioclavicular D scapulothoracic

A

Actions of the proximal and distal radioulnar joint are: A Pronation and supination B Flexion and extension C Circumduction

A, F The superior trunk is composed of fibers from the C5-C6 ventral primary rami. The suprascapular and axillary nerves are composed exclusively of C5-C6 fibers. The long thoracic, musculocutaneous, median, and radial nerves all carry some C5 and C6 fibers and would therefore be partially compromised.

If the superior trunk is injured, which of the following nerves would be completed compromised? (select all that apply) A axillary B lateral pectoral C long thoracic D median E radial F suprascapular

C The musculocutaneous nerve is the nerve of the anterior compartment of the arm.

The muscles of the anterior arm are innervated by which of the following nerves? A Axillary B Median C Musculocutaneous D Radial E Tibial

D

The muscles of the posterior arm are innervated by which of the following nerves? A Axillary B Median C Musculocutaneous D Radial E Tibial

The answer is C: Lateral to the tendon of the flexor carpi radialis. The most common location for measuring pulse rate is on the radial artery at the wrist. Here, the vessel lies on the anterior side of the distal end of the radius, lateral to the tendon of the flexor carpi radialis. It is covered only by skin and a thin superficial fascia and can be palpated eas- ily against the radius. Note the placement of the index and middle fingers of the physician in the given photo. Choice A (Across the anterior aspect of the lateral epicondyle of the humerus) is incorrect. The radial artery originates in the cubital fossa as one of the two terminal branches of the bra- chial artery (the ulnar artery is the other). The radial recur- rent artery branches off the radial artery just below its origin and ascends across the anterior aspect of the lateral epicon- dyle of the humerus. However, the radial recurrent artery lies on muscle and is not normally palpable. Choice B (Between the tendons of the palmaris longus and flexor carpi ulnaris) is incorrect. The ulnar artery enters the hand superficial to the flexor retinaculum, lateral to the pisiform and medial to the hook of the hamate. The ulnar pulse may be palpable slightly lateral to the insertion of the flexor carpi ulnaris onto the pisiform, between it and the tendon of the palmaris longus. Choice D (Superficial to the tendons of the extensor pollicis brevis and abductor pollicis longus) is incorrect. The radial pulse is also available in the anatomical snuffbox, where the radial artery crosses the floor of that space between the ten- dons of the extensor pollicis longus and brevis muscles. In entering the snuffbox, the artery passes deep (not superficial) to the tendons of the abductor pollicis longus and extensor pollicis brevis muscles. Choice E (Superficial to the carpal tunnel) is incorrect. As noted above, the ulnar artery enters the hand superficial to the flexor retinaculum, thus, super- ficial to the carpal tunnel. The superficial palmar branch of the radial artery usually runs through the thenar muscles and is not palpable.

The pulse of the radial artery is readily palpable where the vessel passes which of the following structures? (A)Across the anterior aspect of the lateral epicondyle of the humerus (B) Between the tendons of the palmaris longus and flexor carpi ulnaris (C) Lateral to the tendon of the flexor carpi radialis (D) Superficial to the tendons of the extensor pollicis brevis and abductor pollicis longus (E) Superficial to the carpal tunnel

A Rhomboids retract the scapula.

Which of the following muscles retracts the scapula? A rhomboids B pectoralis minor C serratus anterior D deltoid

E The supraspinatus muscle initiates abduction of the humerus; the deltoid is responsible for completion of the action.

A 19-year-old male is brought to the emergency department with pain in his left shoulder. He reports that the pain started after he felt a pop in his shoulder while playing tennis. Physical exam reveals he is unable to initiate abduction of the arm. An MRI of the shoulder shows a torn muscle. Which muscle was most likely damaged in the injury? A Long head of biceps brachii B Deltoid C Long head of triceps brachii D Trapezius E Supraspinatus

B: Lunate. The lunate is shaped like a moon, thus its name. It is situated in the center of the proximal row of carpal bones where it articulates with the radius. This bone is the most commonly dislocated carpal bone, which leads to severe carpal instabilities. This dislocation often occurs in association with a trans-scaphoid fracture. It is important to note that scaphoid fractures are often difficult to see in radio- graphic imaging; however, these fractures can be detected fre- quently by applying direct pressure to the anatomical snuff- box. Choice A (Scaphoid) is incorrect. The scaphoid bone is located in the proximal row and is the most frequently frac- tured carpal bone. As the weight of the body is transmitted through the upper limb onto the outstretched hand and the impact of the fall exerts pressure back upon the limb, the scaphoid bone is crushed by these opposing forces and is subsequently fractured, usually along its narrowest part. The scaphoid bone is clinically relevant due to the frequency of fractures in younger patients and the poor vascularization of its proximal part. Avascular necrosis is often a postfracture complication that slows the healing of this bone. Choice C (Capitate) is incorrect. The capitate is located in the distal row of carpal bones, so this selection can be easily eliminated. Choice D (Triquetrum) is incorrect. The triquetrum is located in the proximal row of carpal bones and may be involved in a severe dislocation of the wrist. Specifically, the triquetrum can be displaced in Stage III wrist dislocations when the triquetrol- unate interosseous ligament is damaged. However, tearing this interosseous intercarpal ligament would only occur following dislocation of the lunate bone, which is the most commonly dislocated carpal bone. Choice E (Trapezium) is incorrect. The trapezium bone helps form the distal floor of the anatomic snuffbox and forces would be exerted on this bone during the fall. However, it is not located in the proximal row of carpal bones. Moreover, this four-sided bone is rarely fractured or dislocated due to its shape and construction.

A 19-year-old man arrives at his campus health clinic complaining of soreness in his right wrist. He explains he landed on an outstretched hand when he was tackled in a rugby match. He indicates that the pain worsens with movement and is minimized by stabilization of the wrist. There are no sensory deficits in his hand nor does he have trouble grasping or holding objects. Pressure applied to the anatomic snuffbox between the extensor pollicis brevis and extensor pollicis longus tendons produces no pain. Radiographic studies show no fractures but reveal an anterior dislocation of a bone in the proximal row of carpal bones. What carpal bone is most likely dislocated in this patient? (A) Scaphoid (B) Lunate (C) Capitate (D) Triquetrum (E) Trapezium

E The axillary nerve is at risk of injury in shoulder dislocations and in surgical neck fractures. The cutaneous distribution of the axillary nerve is a patch of skin on the lateral aspect of the deltoid muscle.

A 25-year-old male presents to the Urgent Care Clinic with an anterior shoulder dislocation. Nerve injury associated with this dislocation will most likely result in diminished sensation in which of the following locations? A Thumb finger pad B Lateral forearm C Ring finger pad D 1st dorsal web space E Lateral deltoid skin

D

A 34-year-old male comes to the physician because of progressive weakness in his left hand over the past few years. Extensive workup reveals a congenital cervical rib that has compressed the lower trunk of the brachial plexus on his left side. Fibers in which of the following nerves would most likely be affected by the compression of the lower trunk by the cervical rib? A Axillary and thoracodorsal B Lateral cord and medial cord C Musculocutaneous nerve and suprascapular D Ulnar and median E Upper subscapular and lower subscapular

D tingling on the ulnar aspect of the dorsum of the hand indicates an injury superior to the site of origin of the dorsal cutaneous br. of the ulnar nerve in the distal forearm

A 50-year-old patient is brought to the hospital after sustaining a fall from a ladder. The patient complains of severe tingling on the medial aspect of the dorsum and palm of the hand and the volar/palmar surface of the fifth (little) digit. Injury to which of the following nerves is consistent with these findings? A. Superficial radial nerve at wrist B. Median nerve in cubital fossa C. Median nerve in carpal tunnel D. Ulnar nerve at elbow E. Ulnar nerve at wrist

D: Weakness extending the wrist and fingers. The radial nerve is particularly vulnerable during midshaft humeral fractures because it is located directly against the bone in this region. The radial nerve innervates the extensor muscles of the arm and forearm and also carries sensory innervation from the posterior aspect of the arm, forearm, and hand and the lateral aspect of the arm (but not the forearm). Depending upon the exact placement of the fracture, the nerve fibers that inner- vate the triceps brachii may have already left the radial nerve. However, the fibers innervating the extensors of the wrist and fingers would still be bundled in the radial nerve and would be vulnerable to damage during a fracture of this type. Choice A (Numbness on the lateral [radial] aspect of the fore- arm) is incorrect. The radial nerve innervates the skin on the posterior aspect of the arm and forearm and the lateral aspect of the lower arm. The lateral (radial) aspect of the forearm is innervated primarily by the lateral cutaneous nerve of the forearm, the terminal branch of the musculocutaneous nerve. Choice B (Numbness of the medial aspect of the upper arm) is incorrect. The skin over the medial aspect of the upper arm is innervated by the medial cutaneous nerve of the arm (medial brachial cutaneous nerve) that originates from the medial cord of the brachial plexus. This nerve would be protected from the humeral fracture due to its location. Choice C (Numb- ness over the superolateral aspect of the upper arm) is incor- rect. The cutaneous innervation of the superolateral aspect of the upper arm is derived from the axillary nerve that shares a common origin with the radial nerve from the posterior cord of the brachial plexus. Though both nerves arise from the pos- terior cord, the axillary and radial nerves separate proximal to the site of the fracture, thus leaving the axillary nerve undam- aged by the fracture. Choice E (Weakness in grip strength) is incorrect. While the pain associated with the fracture would affect the integrity of all muscle activity in the limb, all of the muscles used for gripping (flexing the wrist and digits) are innervated by the median and ulnar nerves, which do not have close relations to the fracture site.

A 48-year-old woman falls on an icy sidewalk and lands on her right elbow. She suffers a midshaft humeral fracture, as seen on the given X-ray. The attending physician wants to assess whether the nerve residing in the spiral groove of the humerus is damaged. What sign or symptom would confirm damage to this nerve? (A) Numbness on the lateral (radial) aspect of the forearm (B) Numbness of the medial aspect of the upper arm (C) Numbness over the superolateral aspect of the upper arm (D) Weakness extending the wrist and fingers (E) Weakness in grip strength

A the ulnar nerve is the terminal branch of the inferior trunk; the ulnar nerve innervates most of the intrinsic hand muscles including the adductor pollicis and the palmar and dorsal interossei

A 50-year-old woman is diagnosed with an apical lung cancer that invades the inferior trunk of the brachial plexus. Which of the following muscle actions will show the greatest weakness after damage to this nerve? A. Thumb and finger adduction B. Arm abduction C. Elbow flexion D. Thumb extension E. Wrist extension

E the deep radial nerve innervates most of the muscles in the extensor compartment of the forearm; the deep radial nerve has no cutaneous function

A 52-year-old band director suffered problems in her right arm several days after strenuous field exercises for a major athletic tournament. Examination in the orthopedic clinic reveals wrist drop but normal extension of the elbow joint. There is no loss of sensation in the affected limb. Which nerve was most likely affected? A. Deep ulnar B. Median C. Radial D. Superficial radial E. Deep radial

B The displayed deficits are consistent with an injury to the axillary nerve, which can be injured in either a dislocation of the glenohumeral joint or a fracture of the anatomical neck of the humerus.

A 7-year-old boy falls from a tree house and is brought to an emergency room. He has weakness in rotating his arm laterally and sensory loss of the skin over the lateral aspect of the deltoid muscle. Which of the following injuries can result in these functional deficits? Question 2 options: A.Avulsion of the roots of spinal nerves C8 and T1 B.Dislocation of the glenohumeral joint C.Fracture of the clavicle D.Midshaft humerus fracture E.Subluxation of the acromioclavicular joint

A: Ulnar nerve in the elbow. Situations in which peripheral nerves are compressed or otherwise entrapped where they pass through narrow spaces ("tunnels") in muscles and/or osseo-fascial units are generally referred to as "tunnel syndromes." Such conditions may result in periodic or con- stant motor and/or sensory deficits. The ulnar nerve crosses the elbow in a narrow space between the olecranon process and the medial epicondyle of the humerus, on the posteromedial (ulnar) side of the joint. The ulnar nerve can be compressed between these bony landmarks or between the humeral and ulnar heads of the attachment of the flexor carpi ulnaris. Com- pression of the ulnar nerve within these areas leads to "cubital tunnel syndrome." The symptoms are exacerbated during events where flexion of the elbow narrows these passageways and compresses the ulnar nerve at the elbow joint. The patient's cubital tunnel syndrome would explain the paresthesia and numbness on the medial aspect of the hand and the dimin- ished fine motor control of the intrinsic hand muscles. Her condition originated due to the forced flexion of the elbow in the motor vehicle accident, which compressed the ulnar nerve in the cubital tunnel. Choice B (Ulnar nerve in the wrist) is incorrect. The ulnar nerve can be compressed between the pisiform and hook of the hamate at the wrist in a condition termed "ulnar canal syndrome" or "Guyon tunnel syndrome." This entrapment syndrome presents with similar signs and symptoms as seen in this patient. However, the ability to flex the wrist would not be affected. In this patient, trauma to the wrist was not reported. Choice C (Median nerve in the wrist) is incorrect. The median nerve is often compressed at the wrist in "carpal tunnel syndrome" because this nerve travels deep to the transverse carpal ligament (flexor retinaculum of the wrist) within the carpal tunnel. This entrapment syndrome presents with paresthesia of the lateral fingers as well as an inability to oppose the thumb and a wasting of the thenar eminence. These symptoms were not reported in this patient. Choice D (Median nerve in the elbow) is incorrect. The median nerve is not usually compressed by forced flexion of the elbow because it lies loosely on the flexor surface of the joint deep to the bicipital aponeurosis. The median nerve can be compressed in the proximal forearm as it passes between the two heads of the pronator teres. However, the symptoms in this patient were due to forced flexion of the elbow and involve the ulnar nerve. Choice E (Median nerve in the axilla) is incorrect. The median nerve would not be damaged in the axilla without significant trauma. Damage to the median nerve in the axilla would lead to weakness in flexing the wrist, loss of pronation of the forearm, and wasting of the thenar muscles. However, these signs and symptoms were not reported.

A 56-year-old woman was stopped at a light when her car was rear-ended by another car. She had her right arm on the steering wheel, and the impact caused forced flexion at her elbow. Several months later, she comes to her physician complaining of numbness and a "pins and needles" sensation in her right little fi nger when she talks on the phone, rests her head on her right hand at work, or spends most of her day typing at work. She also notices the quality of her typing and her ability to play the violin have diminished. Which nerve is compressed at what location? (A) Ulnar nerve in the elbow (B) Ulnar nerve in the wrist (C) Median nerve in the wrist (D) Median nerve in the elbow (E) Median nerve in the axilla

C Most of the intrinsic muscles of the hand are innervated by the ulnar nerve. This includes the palmar interossei (adductors of the digits) and the dorsal interossei (abductors of the digits).

Assessment of which of the following actions/movements of the digits would provide information regarding the status/integrity of the ulnar nerve? A Using the thumb and index finger to produe an "O" or the "OK sign". B Bringing the tips of the thumb and little finger into contact across the palm. C Successfully resisting removal of a piece of paper being held between two opposed digits. D Extending the wrist against resistance.

E the thenar muscles of the palm are all innervated by the recurrent median n. this nerve is a branch of the median nerve in the palm

Atrophy (decreased motor function with loss of muscle mass) of the thenar muscles is apparent in a patient that presents to your family practice. They also describe numbness and tingling of the thumb, index, and long fingers. These findings are consistent with injury to which of the following nerves? A. deep ulnar B. deep radial C. posterior interosseous D. anterior interosseous E. recurrent median

D the subcromial bursa is located between the head of the humerus and the deltoid muscle

The right shoulder of a 78-year-old female had become increasingly painful over the past year. Abduction of the right arm caused her to wince from the discomfort. Palpation of the deltoid muscle near the acromion process by the physician produced exquisite pain. Imaging studies revealed intermuscular inflammation extending over the head of the humerus. What structure is inflamed? A. Biceps brachii tendon B. Infraspinatus tendon C. Supraspinatus tendon D. Subacromial bursa

A,B,C

Which of the following muscles are responsible for elbow flexion? (select all that apply) A biceps brachii B brachialis C brachioradialis D pronator teres E pronator quadratus F supinator G triceps brachii

B the thoracodorsal n. the innervation of the latissimus dorsi muscle, is a branch of the posterior cord

A patient has been improperly fitted with axillary-type crutches, which have put pressure on the posterior cord of the brachial plexus. Which of the following muscles is most likely to be weakened by this injury? A. Coracobrachialis B. Latissimus dorsi C. Pectoralis minor D. Serratus anterior E. Supraspinatus

B Golfer's elbow is the common term used to describe the medial epicondyle (flexor tendon) tendinopathy

Which of the following listed "conditions" refers to injury of the common flexor tendon at the medial epicondyle? A. Tennis elbow B. Golfer's elbow C. Nursemaid's elbow D. Little leaguer's elbow E. Students bursitis

A,B, C, D A mnemonic for the rotator cuff muscles is "SITS"- supraspinatus, infraspinatus, teres minor, and subscapularis.

Which of the following muscles are part of the rotator cuff? (select all the apply) A subscapularis B supraspinatus C infraspinatus D teres minor E teres major F serratus anterior

C,H

Which of the following muscles are responsible for abduction at the glenohumeral joint? (select all that apply) A biceps brachii long head B coracobrachialis C deltoid D infraspinatus E latissimus dorsi F pectoralis major G subscapularis H supraspinatus I teres major J teres minor

B: Median nerve. Each of the five termi- nal branches of the brachial plexus (musculocutaneous, median, ulnar, radial, and axillary nerves) passes through a muscular or osseofascial tunnel at some point in its distribu- tion, where it may be subject to entrapment in a tunnel syn- drome. The pronator teres muscle arises via two heads, one from the medial epicondyle of the humerus and the other from the coronoid process of the ulna, with a tendinous arch connecting them. The median nerve exits the cubital fossa and enters the forearm by passing between these heads, where it may be unduly compressed in a pronator teres syn- drome. This condition would influence much of the median nerve territory in the forearm plus the entire median nerve territory in the hand. Choice A (Deep branch of the radial nerve) is incorrect. The radial nerve descends from the arm into the cubital fossa, where it divides into superficial and deep branches. The deep branch of the radial nerve pierces the supinator muscle, winds around the proximal end of the radius within the substance of that muscle, and passes into the deep posterior compartment of the forearm as the poste- rior interosseous nerve. The nerve may be entrapped within the supinator, resulting in a supinator syndrome. Such a condition would affect the deeper, more distal extensor muscles arising in the forearm and some sensory areas in the wrist joints. Choice C (Deep branch of the ulnar nerve) is incorrect. The ulnar nerve enters the hand superficial to the flexor retinaculum, runs through a groove between the pisiform and hook of the hamate (Guyon canal), and divides into superficial and deep branches at the base of the hypothenar eminence. The deep branch curls deeply there and enters the deep lying adductor/interosseous compart- ment in the palmar aspect of the hand. Compression of the ulnar nerve in Guyon canal may cause a Guyon tunnel syndrome, which affects the hypothenar muscles, medial two lumbricals, all interossei, adductor pollicis, and a large sensory area on both palmar and dorsal sides of the hand. Choice D (Superficial branch of the ulnar nerve) is incor- rect. The superficial branch of the ulnar nerve does not enter a tunnel and is not subject to a tunnel syndrome. This nerve supplies the palmaris brevis muscle but is mostly cutaneous across the palmar and dorsal aspects of the medial third of the hand. Choice E (Musculocutaneous nerve) is incorrect. This nerve penetrates the coracobrachialis muscle, supplies the three flexor muscles in the anterior compartment of the arm, and continues into the forearm as the lateral cutaneous nerve of the forearm. Entrapment of the nerve within the coracobrachialis is rare.

"Pronator teres syndrome" is a condition in which one of the following nerves is excessively compressed where it passes between the two heads of the pronator teres muscle. Which of the following nerves is entrapped? (A) Deep branch of radial nerve (B) Median Nerve (C) Deep branch of ulnar nerve (D) Superficial branch of ulnar nerve (E) Musculocutaneous nerve

D the tingling sensation is localized to the cutaneous domain of the digital branches of the median nerve

A 50-year-old patient is brought to the hospital after sustaining a fall from a ladder. The patient complains of severe tingling in the palm, and the volar/palmar surface of the index and middle fingers. Injury to which of the following nerves is most consistent with these findings? A. Ulnar B. Radial C. Musculocutaneous D. Median

C: Recurrent branch of median nerve. The recurrent (thenar) branch of the median nerve lies subcuta- neously in the thenar eminence and can be damaged by lac- erations in this area. This nerve innervates most of the thenar muscles, including the Opponens pollicis, Abductor pollicis brevis, and superficial head of the Flexor pollicis brevis (mne- monic = "OAF"). Opposition of the thumb would be lost by cutting the recurrent branch of the median nerve. Though the abductor pollicis brevis is denervated, the abductor pollicis longus, innervated by the deep branch of the radial nerve, is still intact. Also, the flexor pollicis longus, innervated by the anterior interosseous nerve, would still allow flexion at the interphalangeal joint of the thumb. Cutting the recur- rent branch of the median nerve would lead to atrophy of the thenar muscular complex, a condition known as "ape hand." Choice A (Deep branch of the radial nerve) is incorrect. The deep branch of the radial nerve is also called the posterior interosseous nerve as it exits from the supinator muscle. It innervates the abductor pollicis longus and extensor pol- licis longus and brevis muscles, which form the boundaries of the anatomical snuffbox. Cutting this nerve would cause loss of extension of the thumb and weakness in abduction. However, the puncture was on the thenar eminence, and the radial nerve does not travel there. Choice B (Superficial branch of the radial nerve) is incorrect. The superficial branch of the radial nerve arises from the radial nerve in the cubital fossa. This nerve is purely cutaneous, supplying sensation to the dorsum of the hand and fingers. Because it does not have a motor component, this nerve would not be responsible for the deficit in thumb function. Choice D (Deep branch of ulnar nerve) is incorrect. The deep branch of the ulnar nerve innervates most of the intrinsic muscles of the hand, including the hypothenar muscles, medial two lumbrical muscles, the palmar and dorsal interossei, adductor pollicis, and the deep head of the flexor pollicis brevis. With a small motor supply to the thenar compartment (specifically the deep head of the flexor pollicis brevis), this nerve could not cause a substantial loss of function in the thenar eminence. Choice E (Superficial branch of the ulnar nerve) is incorrect. The superficial branch of the ulnar nerve arises from the ulnar nerve distal to the flexor retinaculum. This nerve supplies cutaneous branches to the anterior surface of the medial one and a half fingers. The palmaris brevis is the only muscle supplied by this nerve, and this muscle tightens the skin of the medial surface of the palm. Therefore, this nerve would not be responsible for substantial loss of motor function in the thumb.

A 10-year-old boy was running across a parking lot when he tripped and received lacerations on the base of his thumb from a broken glass bottle. On examination, his thumb was unable to oppose to his fingers, and the thumb also showed weakness when abducting and flexing. No sensory deficits were reported. What nerve was most likely severed? (A) Deep branch of radial nerve (B) Superficial branch of radial nerve (C) Recurrent branch of median nerve (D) Deep branch of ulnar nerve (E) Superficial branch of ulnar nerve

C. the boy's symptoms are consistent with a radial nerve injury. The fact that he can extend his elbow indicates that the nerve has been injured distal to its branches to the triceps brachii muscle, which occurs proximally in the arm

A 13-year-old boy is brought to the emergency department after crashing in a motorbike race. His left arm was run over by another racer's motorbike. He is unable to extend the left wrist, fingers, and thumb, although he can extend the elbow. He also experiences loss of sensation in the lateral half of the dorsum of the left hand. Which of the following nerves has most likely been injured to result in these signs, and in what part of the upper limb is the injury located? A. Median nerve, anterior wrist B. Median nerve, arm C. Radial nerve, midhumerus D. Ulnar nerve, midlateral forearm E. Ulnar nerve, midpalmar region

E the patient is displaying the classic signs of an upper trunk injury affecting the suprascapular, axillary, and the musculocutaneous nerve

A 16-year-old male is brought to the Emergency Department after being tackled hard during a football game. He says that he collided with an opposing player and was hit very hard on the neck and shoulder by his opponent's helmet. On physical examination, the patient's right arm hangs at his side and is medially rotated with his forearm pronated. He is unable to abduct the affected arm. Which of the following is the most likely site of the lesion causing this patient's symptoms? A. Long thoracic nerve B. Proximal C8-T1 trunk of the brachial plexus C. Proximal posterior cord of the brachial plexus D. Ulnar nerve just distal to the medial epicondyle of the humerus E. Upper trunk of the brachial plexus

B: Torn coracoclavicular ligament. "Shoulder separation" describes a dislocation of the acromioclavicular joint. In its most severe form (grade 3), the condition includes a tearing of both the intrinsic acromioclavicular ligament and the extrinsic coracoclavicular ligament. As a result, the scapula separates from the clavicle and falls away due to the weight of the upper limb. Thus, the distal end of the clavicle is promi- nent. Choice A (Dislocated head of the humerus) is incorrect. Dislocations of the GH joint easily occur inferiorly due to its lack of muscular and ligamentous support. Thus, damage to the axillary nerve often occurs following inferior displacment of the head of humerus from the GH joint. However, the acro- mioclavicular joint, which is more proximal, was injured in this patient. Dislocations of the glenohumeral joint in other directions are more difficult (but not impossible) because of the support of the rotator cuff muscles (anteriorly and pos- teriorly) and the coracoacromial arch (superiorly). Choice C (Fractured clavicle) is incorrect. Radiological imaging would have detected a fractured clavicle, but these tests confirmed a shoulder separation and not a fractured clavicle. Choice D (Dislocated sternal end of the clavicle) is incorrect. Due to its intrinsic strength, dislocation of the sternoclavicular (SC) joint is rare. Most dislocations of the SC joint occur in persons less than 25 years of age following a fracture of the epiphysial plate of the clavicle. The epiphysis at the proximal end of the clavicle does not close until approximately age 25. Though this patient was under the age of 25, his injury was localized to the acromioclavicular joint. Choice E (Torn anterior gleno- humeral [GH] ligament) is incorrect. Three GH ligaments reinforce the anterior part of the joint capsule; however, the GH joint was not involved in this patient.

A 17-year-old boy comes to the emergency room after a hard fall onto the lateral aspect of his left shoulder during a high school basketball game. He complains of generalized pain during shoulder motion. On physical examination, the distal end of the clavicle is prominent and distinctly palpable. Radiological findings confirm the diagnosis of a severe (grade 3) shoulder separation. Which of the following features is a component of this condition? (A) Dislocated head of the humerus (B) Torn coracoclavicular ligament (C) Fractured clavicle (D) Dislocated sternal end of the clavicle (E) Torn anterior glenohumeral (GH) ligament

A: Axillary nerve. The axillary nerve may be damaged in approximately one of seven shoulder dislocations. This nerve innervates the deltoid and teres minor muscles as well as supplying innervation to the skin overlying the deltoid in the superolateral aspect of the arm. Loss of innervation to the deltoid muscle would explain the weakness in abduction of the upper limb. The teres minor assists the infraspinatusmuscle in external rotation of the shoulder. Choice B (Median nerve) is incorrect. The median nerve does not branch proxi- mal to the elbow, and its sensory distribution is limited to the hand. The median nerve could not be responsible for the patient's motor and sensory deficits. Choice C (Ulnar nerve) is incorrect. The ulnar nerve can be damaged during shoul- der dislocations; however, its sensory distribution is limited to distal to the wrist, and the first muscle it innervates is in the forearm (flexor carpi ulnaris). Therefore, the ulnar nerve could not be responsible for this patient's motor and sensory deficits. Choice D (Radial nerve) is incorrect. The radial nerve supplies motor innervation to the posterior compartments of the arm and forearm. Damage to this nerve would cause weakness in extending at the elbow and wrist joints. The radial nerve gives rise to the posterior cutaneous nerves of the arm and forearm as well as the inferior lateral cutaneous nerve of the arm. However, it would not affect the superior lateral cutaneous nerve of the arm, arising from the axillary nerve, which was damaged in this patient. The radial nerve would also not affect abduction and external rotation of the shoulder. Choice E (Musculocutaneous nerve) is incorrect. The muscu- locutaneous nerve can be damaged during shoulder disloca- tions; however, this nerve supplies the motor innervation to the anterior compartment of the arm. Trauma to this nerve would lead to weakness in flexing the elbow and supinating the forearm. Its sensory distribution is limited to the lateral aspect of the forearm, so it was not the nerve damaged in this patient.

A 17-year-old male football player suffers a shoulder injury and arrives at the ER 2 hours after the injury. The physician diagnoses a shoulder dislocation, and after administration of a local anesthetic solution, the doctor repositions the head of the humerus into the glenoid cavity of the scapula (reduction). No fractures are seen on X-rays. However, the patient displays weakness in abduction and external rotation at the shoulder. A loss of sensation is also noted at the superior and lateral aspects of the arm. What nerve was most likely damaged in this injury? (A) Axillary nerve (B) Median nerve (C) Ulnar nerve (D) Radial nerve (E) Musculocutaneous nerve

A Upper trunk of the brachial plexus. The illustration shows an injury in which the cervicobrachial angle (the angle between the neck and shoulder) is stretched widely. This abnormal impact eventually results in the postural pre- sentation of a "waiter's tip" deformity (Erb-Duchenne palsy). This combination of injury and postural deformity is related to damage to both the C5 and C6 roots or upper trunk of the brachial plexus. The C5 and C6 roots converge to form the upper trunk of the brachial plexus and contribute heavily to the suprascapular, axillary, and musculocutaneous nerves. The suprascapular nerve supplies the supraspinatus and infraspi- natus muscles. The axillary nerve controls the deltoid and teres minor muscles. The musculocutaneous nerve supplies the anterior compartment of the arm (coracobrachialis, biceps brachii, brachialis muscles). Therefore, a significant weakness in abduction and lateral rotation of the shoulder, flexion of the shoulder and elbow, and supination of the forearm would result from this brachial plexus injury. The ultimate postural deformity is a contracture effect in which the intact muscles act unopposed to draw the limb into a position that is the oppo- site of the actions of the affected muscles. Choice B (Lower roots of the brachial plexus) is incorrect. The lower roots (C8, T1) have a strong projection into the ulnar nerve. Trauma here would result in an ultimate postural deformity of "claw hand" due to the loss of flexion of the medial digits. That injury does not match this patient's clinical presentation. Choice C (Poste- rior divisions of the brachial plexus) is incorrect. The posterior divisions supply the radial, axillary, upper and lower subscap- ular, and thoracodorsal nerves. The primary postural effect resulting from trauma here would be a case of "wrist drop" expressed due to loss of the extensor muscles innervated by the radial nerve. Choice D (Medial cord of the brachial plexus) is incorrect. The medial cord projects into the ulnar and median nerves. Damage to the ulnar nerve will result in "claw hand." The classic median nerve deformity of "ape hand" probably would not be realized because the median nerve also receives a strong input from the lateral cord. Choice E (Lateral root of the median nerve) is incorrect. The median nerve is formed by lateral and medial roots derived from the lateral and medial cords, respectively. Trauma to either root would weaken the territory of the median nerve but likely not result in the classic median nerve postural defect of "ape hand." The lateral root does not contribute to the suprascapular and axillary nerves, which have been affected in this patient.

A 50-year old man falls off a ladder while cleaning his windows, landing on the ground as seen in the given drawing. He does not seek medical aid, believing his general soreness will go away with time. However, after several months, he develops a postural deformity of his left upper limb that includes an adducted, medially rotated, and extended shoulder, extended elbow, and pronated forearm. The injury and subsequent condition reflect damage to what structure? (A) Upper trunk of the brachial plexus (B) Lower roots of the brachial plexus (C) Posterior divisions of the brachial plexus (D) Medial cord of the brachial plexus (E) Lateral root of the median nerve

E: Fracture of the fifth metacarpal bone. Fracture of the fifth metacarpal bone is called a "boxer's fracture" because this injury is often seen after an individual improp- erly punches a solid object with a clenched fist. The impact on the head of the fifth metacarpal causes the distal shaft of this bone to fracture. Skilled pugilists are trained to direct the impact of the clenched fist on the heads of the first and second metacarpals to avoid this type of injury. Choice A (Disloca- tion of the fifth metacarpophalangeal joint) is incorrect. This type of injury is rare, and a dislocation of the fifth metacarpo- phalangeal joint would have been detected by the physician on examination. Choice B (Fracture of the triquetral bone) is incorrect. This carpal bone is rarely fractured in this type of impact. Choice C (Fracture of the proximal phalanx of the ring finger) is incorrect. The impact of the clenched fist would have been localized to the head of the fifth metacarpal. Damage to the proximal phalanx of the ring finger would have occurred only if the fist was not clenched during impact. Choice D (Fracture of the proximal phalanx of the little fin- ger) is incorrect. The impact of the clenched fist would have been localized to the head of the fifth metacarpal. Damage to the proximal phalanx of the little finger would have occurred only if the fist was not clenched during impact.

A 17-year-old man has pain and moderate swelling over the dorsomedial aspect and in the hypothenar area of his right hand after punching a locker over a dispute with his girlfriend. What is the most likely finding on an X-ray of his hand? (A) Dislocation of the fifth metacarpophalangeal joint (B) Fracture of the triquetral bone (C) Fracture of the proximal phalanx of the ring finger (D) Fracture of the proximal phalanx of the little finger (E) Fracture of the fifth metacarpal bone

B The most commonly fractured carpal bone is the scaphoid that forms the floor of the ana- tomical snuffbox. This area is a fossa located between the three long tendons of the thumb (tendons of abductor pollicis lon- gus and extensor pollicis brevis laterally and tendon of the extensor pollicis longus medially). The scaphoid bone is fre- quently broken when an individual falls with an outstretched hand and lands on the palm with the hand abducted. A broken scaphoid bone is commonly seen in individuals under the age of 30. The scaphoid is broken due to its unfortunate position between the downward force transmitted by the weight of the upper limb and the upward force due to the impact of hitting the floor. The proximal aspect of a broken scaphoid bone can suffer from avascular necrosis due to its blood supply enter- ing the bone distally. Choice A (Capitate) is incorrect. The capitate is located centrally in the distal row of carpal bones and articulates with most of the carpal bones (the triquetrum, pisiform, and trapezium being the exceptions). This bone is not located in the floor of the anatomical snuffbox, so it was not the most likely bone injured in this patient. Choice C (Hamate) is incorrect. The hamate is located in the distal row of carpal bones on the ulnar side. It is not related to the ana- tomical snuffbox, so it would not produce pain in the base of the thumb when broken. Choice D (Trapezium) is incorrect. The trapezium is located at the base of the thumb, but it is not commonly fractured when a person falls with an outstretched hand. A mnemonic for the position of the trapezium is "tra- pezium articulated with the thumb." Choice E (Pisiform) is incorrect. The pisiform is located in the proximal carpal row and is a sesamoid bone located within the tendon of the flexor carpi ulnaris muscle. Its position on the ulnar side of the wrist would not cause pain in the anatomical snuffbox.

A 21-year-old man goes to his college campus health clinic complaining of soreness in his left wrist after falling on an outstretched hand during a basketball game the previous day. He is supporting his left wrist and indicates that the pain worsens with movement and is minimized with inactivity. There is no loss of feeling in his hand, nor does he have trouble grasp- ing or holding objects. The physician exacerbates the wrist pain by applying pressure to the base of the thumb in the anatomical snuffbox (see photo). Radiographic imaging will confirm a break of which carpal bone? (A) Capitate (B) Scaphoid (C) Hamate (D) Trapezium (E) Pisiform

B The ulnar nerve innervates the hypothenar muscles and most of the other intrinsic hand muscles and carries sensations from the medial 1 1/2 fingers (the little finger and the medial 1/2 of the ring finger). The median nerve innervates the muscles of the thumb and skin over the lateral 3 1/2 fingers. The axillary nerve, radial nerve, and musculocutaneous nerves do not innervate any of the hand muscles. The radial nerve innervates the skin on the posterior surface of the hand only.

A 23 year old female college student comes to the clinic complaining of a weak grip and loss of sensation on the medial 1 1/2 fingers (little finger and medial 1/2 of ring finger). A pin prick test to the skin of her finger tips confirms that she cannot feel touch sensations on her medial 1 1/2 fingers. Damage to which of the following nerves explains her medical condition? A median B ulnar C radial D musculocutaneous E axillary

The answer is D: Biceps brachii. The lateral X-ray reveals chronic microtrauma to the radial tuberosity, which is the insertion site for the biceps brachii muscle. In this weight- lifter, the damage to the radial tuberosity is most likely due to overuse of the biceps brachii and the concomitant stress placed upon the radial tuberosity by lifting significant weight loads. The biceps brachii muscle produces flexion of the elbow (and shoulder) and is the powerful supinator of the forearm, explaining why these actions exacerbated the pain in this patient. Choice A (Supinator) is incorrect. The supinator muscle attaches onto the proximal shaft of the radius rather than the radial tuberosity, so it would not cause damage to the radial tuberosity. Moreover, the supinator muscle acts to pro- duce lower resistance supination and does not flex the elbow. Choice B (Brachialis) is incorrect. The brachialis muscle is a powerful flexor of the elbow. However, it is not related to the radial tuberosity as it attaches onto the coronoid process and tuberosity of the ulna. Furthermore, the brachialis muscle does not act in supination. Choice C (Pronator teres) is incorrect. The pronator teres muscle contributes somewhat to flexion of the elbow. However, it inserts onto the midshaft portion of the radius, and its main action is pronation. In this patient, pain was exacerbated by supination against resistance. Choice E (Brachioradialis) is incorrect. The brachioradials muscle is a notable flexor of the elbow. However, it attaches to the styloid process at the distal end of the radius and is not related to the radial tuberosity. Also, the brachioradialis muscle does not act in supination.

A 23-year-old competitive weight lifter goes to his physician complaining of pain in his proximal forearm. During his examination, the pain is exacerbated by flexion of the elbow and supination of the forearm against resistance. A lateral radiograph shows chronic microtrauma to the proximal radius, marked by the black arrows. Which of the following muscles attaches to, and most likely damaged, this osteological process? (A) Supinator (B) Brachialis (C) Pronator teres (D) Biceps brachii (E) Brachioradialis

D: Abduction of the index finger. The sensory deficit occurs in the cutaneous territory of the ulnar nerve. This nerve controls most of the intrinsic muscles of the hand, including the first dorsal interosseous muscle, which controls abduction of the index finger. The four dorsal interosseous muscles of the hand are innervated by the deep branch of the ulnar nerve and function to abduct digits 2 to 4. Remember that two fingers (the thumb and little finger) have their own muscles dedicated to abduction. Also, remember the mne- monic "DAB," which stands for Dorsal interossei ABduct the fingers. Thus, the associated motor deficit for the ulnar nerve on this list would be loss of abduction of the index finger. Choice A (Pronation) is incorrect. Pronation is governed by the median nerve, which supplies the pronator teres and pronator quadratus muscles. Choice B (Abduction of the wrist) is incor- rect. Abduction of the wrist is controlled by the median nerve (supplying the flexor carpi radialis) and the radial nerve (sup- plying the extensor carpi radialis longus and brevis). Choice C (Extension of the wrist) is incorrect. Extension of the wrist is produced by the radial nerve acting mainly on the extensor carpi radialis longus and brevis and the extensor carpi ulnaris muscles. Choice E (Flexion of the interphalangeal joints of the index finger) is incorrect. Flexion of the interphalangeal joints of the index finger is controlled by branches of the median nerve, which supply the flexor digitorum superficialis muscle (to flex the proximal interphalangeal joint of the second fin- ger) and the flexor digitorum profundus muscle (to flex the distal interphalangeal joint of the index finger).

A 23-year-old medical student complains of loss of sensation in the skin on the medial edge of her left hand, including the entire fifth digit. The associated motor deficit probably involves weakness in which of the following? (A) Pronation (B) Abduction of the wrist (C) Extension of the wrist (D) Abduction of the index finger (E) Flexion of the interphalangeal joints of the index finger

E: Anular ligament of radius. The anular ligament of the radius encircles and holds the head of the radius in the radial notch of the ulna (see diagram on next page). This ligament enables pronation and supination of the forearm. However, the head of the radius can be pulled dis- tally out of this anular ligament resulting in a subluxation or dislocation of the radial head, which is frequently called "nursemaid's elbow." This injury is often seen in children, particularly girls, between the ages of 1 to 3 years old. It occurs when an extended arm is pulled, commonly during a fall, and the individual holding the hand does not let go, as reported in this case. Subluxation and dislocation of the radial head are also seen when the child is swinging while being held by the hands. Choice A (Interosseous membrane of the forearm) is incorrect. The interosseous membrane of the forearm connects the interosseous borders of the radius and ulna. This membrane would not be injured in this type of the injury, especially because the patient complains of pain in the elbow region. Choice B (Quadrate ligament) is incor- rect. The quadrate ligament passes from the distal margin of the radial notch of the ulna to the neck of the radius. Dam- age to this ligament is rare in this type of injury when the extended arm is pulled. Choice C (Radial collateral ligament of elbow) is incorrect. The radial collateral ligament of the elbow extends from the lateral epicondyle of the humerus to the anular ligament of the radius. This ligament would be injured in forced adduction of the elbow joint, which was not seen in this patient. Choice D (Ulnar collateral ligament of elbow) is incorrect. The ulnar collateral ligament of the elbow extends from the medial epicondyle of the humerus to the coronoid process and olecranon of the ulna. This lig- ament would be injured in forced abduction of the elbow joint, which was not seen in this patient. This ligament is also frequently injured in athletes who use an overhead throwing motion. When the athlete releases the ball during their throw- ing motion, the ulna is pulled from the humerus, which can damage this ligament. American baseball pitchers often need surgery to repair the ulnar collateral ligament of the elbow, which is commonly termed "Tommy John surgery."

A 3-year-old girl is brought to the emergency room holding her right arm with the elbow flexed and the forearm pronated. She refuses to move her arm and complains her elbow "hurts a lot." Her mother reports they were holding hands and running in the park when the child tripped. The mother pulled on the child's hand to prevent her from hitting the ground. Given the nature of this injury and the age of the patient, what structure is most likely damaged? (A) Interosseous membrane of forearm (B) Quadrate ligament (C) Radial collateral ligament of elbow (D) Ulnar collateral ligament of elbow (E) Anular ligament of radius

E: Superficial branch of the radial nerve. The superficial branch of the radial nerve is entirely cutaneous, carrying sensation from the dorsolateral part of the hand from the anatomical snuffbox to the midline of the fourth finger. This nerve is vulnerable as it runs posteriorly between the bra- chioradialis and extensor carpi radialis longus tendons toward the dorsum of the hand. This nerve was damaged at this loca- tion, but the area of sensory loss is less than expected due to the overlap from cutaneous branches of the ulnar and median nerves. Choice A (Dorsal cutaneous branch of the ulnar nerve) is incorrect. The dorsal cutaneous branch of the ulnar nerve passes posterior between the ulna and flexor carpi ulnaris to supply the subcutaneous tissue of the dorsal aspect of the pos- teromedial aspect of the hand, medial to the midline of the fourth finger. Its medial location and its sensory distribution make it an unlikely choice to be involved with this patient.Choice B (Lateral cutaneous nerve of the forearm) is incorrect. The lateral cutaneous nerve of the forearm is the continuation of the musculocutaneous nerve, which exits the arm between the biceps brachii and brachialis muscles. It supplies the skin on the lateral aspects of the forearm and wrist but would not be responsible for the numbness and paresthesia seen in this patient because the sensory deficit is distal to this nerve's normal distribution. Choice C (Posterior cutaneous nerve of the forearm) is incorrect. The posterior cutaneous nerve of the forearm arises from the radial nerve and passes in close prox- imity to the lateral intermuscular septum of the arm near the origin of the brachioradialis. As its name implies, it supplies the posterior aspect of the forearm, but it does not extend past the wrist. Therefore, it would not be the nerve damaged in this patient. Choice D (Deep branch of the radial nerve) is incorrect. The deep branch of the radial nerve is entirely motor in its distribution to the posterior muscles of the fore- arm. Its lack of cutaneous innervation makes this choice easy to eliminate.

A 36-year-old man broke a window with his fi st to rescue his child from a house fi re. The man sustained a laceration to the lateral aspect of his right forearm, but he only showed a sensory defi cit (numbness and paresthesia) to the dorsolateral aspect of his hand (as denoted by the shaded area within the given photo). What nerve was most likely damaged? (A) Dorsal cutaneous branch of the ulnar nerve (B) Lateral cutaneous nerve of the forearm (C) Posterior cutaneous nerve of the forearm (D) Deep branch of the radial nerve (E) Superficial branch of the radial nerve

E: Scaphoid. The proximal segment of the scaphoid bone has a poor supply of blood because the pal- mar carpal branch of the radial artery enters the distal part of the scaphoid and then supplies blood proximally. This small artery is often severed during fractures of the scaphoid bone leading to avascular necrosis, or death of the bone due to poor blood supply. Remember, the scaphoid is the most commonly fractured carpal bone, but a fracture within this bone is often not seen on initial radiographs. Radiographs taken several weeks later will show the fracture due to bone resorption at the fracture site. Choice A (Distal radius) is incorrect. The distal radius is often fractured in older individuals who fall on their outstretched hand. This fracture of the distal radius is called a Colles fracture ("dinner fork deformity") due to its appearance on a lateral radiograph. However, the distal radius has an adequate blood supply and is not prone to avascular necrosis. Choice B (Midshaft ulna) is incorrect. A fracture of the midshaft ulna is not prone to avascular necrosis due to its adequate blood supply. Choice C (Fifth metacarpal) is incorrect. The fifth metacarpal is often fractured when an indi- vidual improperly punches a solid object with a clenched fist, as in a "boxer's fracture." This bone is not prone to avascular necrosis due to its adequate blood supply. Choice D (Lunate) is incorrect. The lunate is the most commonly dislocated carpal bone, which leads to severe carpal instabilities. This dislocation often occurs in association with a trans-scaphoid fracture. If this bone were fractured in this work-related acci- dent, it would not be prone to avascular necrosis due to its adequate blood supply.

A 37-year-old factory worker fractures multiple bones distal to the elbow when his hand and forearm are crushed by equipment dropped by a faulty hydraulic lift. Which of the following bones, if fractured, would most likely develop avascular necrosis? (A) Distal radius (B) Midshaft ulna (C) Fifth metacarpal (D) Lunate (E) Scaphoid

C the sensory deficits are consistent with an ulnar nerve injury. The palmar and dorsal interossei are innervated by the deep ulnar n.

A 37-year-old woman presents to her physician complaining of a loss of sensation in the skin on the medial edge of her left hand, including the entire fifth digit. Deficits in which of the following actions are expected in this patient? Question 13 options: A. Abduction of the wrist B. Extension of the wrist C. Abduction of the index finger D. Flexion of the proximal and distal interphalangeal joints of the index finger

B: Lower trunk of the brachial plexus. This woman has experienced a lower brachial plexus injury due to forced abduction of the upper limb during the accident. This injury presents with numbness and paresthesia in the C8 and T1 dermatomes, which supply the axilla and medial aspect of her upper limb. These nerve roots primarily supply the medial cord of the brachial plexus, which creates the ulnar nerve. Due to damage to the ulnar nerve, she is experiencing weakness in the movement of her left hand. The abduction and adduction of the fingers are controlled by the deep branch of the ulnar nerve by supplying the dorsal interosseous and palmar interosseous muscles, respectively. Choice A (Upper trunk of the brachial plexus) is incorrect. Damage to the upper trunk of the brachial plexus results in Erb-Duchenne palsy ("waiter's tip malformation"). In this injury, the patient presents with significant weakness in abduction and lateral rotation of the shoulder, flexion of the shoulder and elbow, and supination of the forearm. Choice C (Posterior cord of the brachial plexus) is incorrect. The posterior cord of the brachial plexus gives rise to the axillary and radial nerves. Damage to the axillary nerve causes weakness in abduction of the shoulder due to loss of the deltoid muscle. Damage to the radial nerve results in inability to extend at the elbow or wrist (leading to "wrist drop"). However, this patient had significant weakness in the movements of the hand, which implies damage to the contributions of the ulnar nerve. Choice D (Lateral cord of the brachial plexus) is incorrect. Damage to the lateral cord of the brachial plexus results in signs and symptoms similar to those seen in Erb-Duchenne palsy, or the "waiter's tip malformation." In this injury, the patient presents with significant weakness in abduction and lateral rotation of the shoulder, flexion of the shoulder and elbow, and supination of the forearm. Choice E (Long tho- racic nerve) is incorrect. A lesion of the long thoracic nerve leads to a "winged scapula" due to the subsequent paralysis of the serratus anterior muscle. Additionally, the affected arm cannot be abducted above the horizontal plane because the serratus anterior is not available to superiorly rotate the gle- noid cavity of the scapula to allow full abduction. This deficit was not seen in this patient.

A 50-year female equestrian is thrown from a startled horse and dragged by the reins, which were wrapped around her left wrist, for some distance. At the ER, she is experiencing pain and paresthesia in the axilla and medial aspect of her upper limb. Despite being left-handed, she has marked weakness in the movements of her dominant hand, especially abduction and adduction of the fingers. What structure was most likely damaged in this woman? (A) Upper trunk of the brachial plexus (B) Lower trunk of the brachial plexus (C) Posterior cord of the brachial plexus (D) Lateral cord of the brachial plexus (E) Long thoracic nerve

B: Latissimus dorsi. The axilla is a large, pyra- midal space between the side of the chest and the upper part of the brachium. Its major importance is as a passageway from the root of the neck to the upper limb. The axilla is demarcated by four walls: anterior, posterior, medial, lateral. The posterior wall is composed of the latissimus dorsi, teres major, and subscapularis muscles. The posterior axillary fold forms the palpable lower margin of the wall and is composed of the latissimus dorsi and teres major. The subscapularis is not part of the posterior axillary fold. Damage to the latis- simus dorsi would severely hinder adduction, extension, and medial rotation of the arm. Choice A (Pectoralis minor) is incorrect. The pectoralis minor contributes to the formation of the anterior wall of the axilla. The pectoralis major forms the bulk of the anterior wall and creates the noticeable ante- rior axillary fold. Choice C (Levator scapulae) is incorrect. The levator scapulae muscle passes out of the neck to attach onto the superior angle of the scapula. It is far removed from any of the walls of the axilla and does not contribute to rota- tion and adduction of the arm. Choice D (Serratus anterior) is incorrect. The serratus anterior lies against the thoracic wall and forms much of the medial wall of the axilla along with the thoracic wall. It fixes the scapula to the thoracic wall and has no affect on rotation of the arm. Choice E (Teres minor) is incorrect. The teres minor is located immediately above the teres major. However, it is not a component of the posterior wall of the axilla and normally contributes to lateral rotation of the arm.

A 65-year-old man is brought to the emergency room after being attacked in his office by a disgruntled co-worker. The attacker reportedly used a long, narrow-bladed letter-opener to inflict multiple stab wounds to the man's back. Physical examination shows a puncture wound in the posterior axillary fold. The patient presents with weakness in extension, adduction, and medial rotation of his arm. Which of the following muscles is most likely cut in this injury? (A) Pectoralis minor (B) Latissimus dorsi (C) Levator scapulae (D) Serratus anterior (E) Teres minor

A These are the classic symptoms of carpal tunnel syndrome. Additional functional deficits in digit flexion and/or pronation would suggest an injury of the median nerve more proximally.

A 69-year-old woman visits the outpatient clinic with a complaint of numbness and tingling of her hand for the past 3 months. Physical examination reveals she has numbness and pain in the lateral three digits of her right hand that are relieved by vigorous shaking of the wrist. In addition, opposition of the thumb is weakened relative to the left hand. Sensation was decreased over the lateral palm and the volar aspect of the first three digits. No other functional deficits in the limb were observed. These symptoms are consistent with compression of which nerve at which location? A Median nerve in the carpal tunnel. B Ulnar nerve at the elbow (between the two heads of flexor carpi ulnaris). C Median nerve between the two heads of pronator teres. D Radial nerve in the radial groove (due to a poorly fitted crutch). E Inferior trunk in the interscalene triangle.

D. radial artery The radial and ulnar arteries anastomose to form the superficial and deep palmar arches. However, the superficial palmar arch is formed mainly by the ulnar artery, whereas the deep palmar arch is formed primar- ily by the radial artery. Thus, the insufficiency in this case is most likely due to reduced flow in the radial artery. Choice A (Posterior interosseous artery) is incorrect. The posterior interosseous artery branches off the common interosseous artery, which is derived from the ulnar artery in the proxi- mal forearm. The posterior interosseous artery supplies the posterior compartment of the forearm. This artery does not reach the wrist and does not contribute blood to the palmar arterial arches. Choice B (Ulnar artery) is incorrect. The ulnar artery provides the primary supply into the superficial pal- mar arterial arch. The deep palmar arch is formed primarily by the radial artery, and it is the reduced flow of the radial artery most likely present in this patient. Choice C (Anterior interosseous artery) is incorrect. The anterior interosseous artery branches off the common interosseous artery, which is derived from the ulnar artery in the proximal forearm. The anterior interosseous artery supplies the anterior compart- ment of the forearm as well as the distal posterior forearm via its posterior terminal branch. This artery does reach the wrist, but it does not contribute blood to the palmar arterial arches. Choice E (Inferior ulnar collateral artery) is incorrect. This artery branches off the brachial artery just above the elbow. It passes distally across the anterior side of the medial epicon- dyle of the humerus to form a collateral connection with the anterior ulnar recurrent branch of the ulnar artery. This vessel 32 is far removed from the hand.

A 74-year-old man complains of pain in his right hand and fingers when he works with his hands for a while. Thorough testing reveals insufficient blood fl ow into the deep palmar arch. Occlusion of which of the following arteries is the most likely cause of this condition? (A) Posterior interosseous artery (B) Ulnar artery (C) Anterior interosseous artery (D) Radial artery (E) Inferior ulnar collateral artery

D the palmar/volar surface of the little finger is innervated by the ulnar nerve A- radial n. B-median n. C-median n. D- ulnar n. E- lat. cutaneous br from musculocutaneous n.

A patient presents to the emergency department with a medial epicondylar fracture. Ulnar nerve damage in this injury is confirmed by decreased sensation at which of the following locations? A. dorsum of the first and second web space B.digit pad of the index finger C. digit pad of the middle finger D. digit pad of the little finger E. lateral forearm

C: Long thoracic nerve. The photo demonstrates a case of "winged scapula," indicative of a lesion of the long thoracic nerve and subsequent paralysis of the serratus anterior muscle. The nerve runs down the lateral thoracic wall, on the superficial aspect of the serratus anterior, where it is unusually exposed (for a motor nerve) and vulnerable to injury, especially when the limb is elevated. Lesion of the nerve denervates the serratus anterior. This results in the medial border and inferior angle of the scapula pulling away from the posterior chest wall, giving the scapula a wing-like appearance when the affected limb is protracted. Additionally, the affected arm cannot be abducted above the horizontal plane because the serratus ante- rior is not available to superiorly rotate the glenoid cavity of the scapula to allow full abduction. Choice A (Axillary nerve) is incorrect. This nerve passes deeply through the axilla, around the surgical neck of the humerus, to supply the teres minor and deltoid muscles. Lesion here would result in significant weak- ness in abduction of the arm and wasting of the rounded con- tour of the shoulder. Choice B (Thoracodorsal nerve) is incor- rect. This nerve runs inferior through the axilla to supply the latissimus dorsi muscle. Loss of the nerve would result in weak- ness in extension and medial rotation of the arm, plus wasting of the posterior axillary fold. Choice D (Dorsal scapular nerve) is incorrect. This nerve courses into the upper, medial part of the back and the lower neck to supply the levator scapulae and rhomboid muscles. Paralysis of these muscles would result in weakness in elevation and retraction of the scapula and perhaps wasting of the contour of the back under the trapezius muscle. Choice E (Suprascapular nerve) is incorrect. The suprascapu- lar nerve runs through the suprascapular notch and into the supraspinous and infraspinous fossae to supply the supraspina- tus and infraspinatus muscles. Lesion of this nerve would result in weakness in the rotator cuff affecting the initiation of abduc- tion (supraspinatus) and external rotation (infraspinatus) of the shoulder and wasting of the muscular contour of the posterior aspect of the scapula.

An 18-year-old boy is cut severely on the lateral wall of his right chest during a knife fight. Following healing, his scapula moves away from the thoracic wall when he leans on his right hand, giving the appearance indicated in the given photo. Which of the following nerves is likely damaged? (A) Axillary nerve (B) Thoracodorsal nerve (C) Long thoracic nerve (D) Dorsal scapular nerve (E) Suprascapular nerve

C The teres minor and infraspinatus muscles are lateral rotators of the humerus. Supraspinatus abducts the humerus; latissimus dorsi, teres major and subscapularis medially rotate the humerus.

An 18-year-old male presents to the emergency department following an accident. Examination reveals weakened ability to laterally rotate his arm. Which of the following muscles is most likely affected by this injury? A Latissimus dorsi B Teres major C Infraspinatus D Subscapularis E Supraspinatus

A The anterior interosseous branch of the median nerve innervates the flexor pollicis longus, the radial half of flexor digitorum profundus and the pronator quadratus. The first two of these muscles produce the flexion of the IP (Thumb) and DIP joints (index finger) necessary to produce the "O". Note that the anterior interosseous nerve has no cutaneous representation. As a consequence motor deficits in the absence of sensory deficits indicates an injury localized to the anterior interosseous nerve. Observation of sensory deficits as well would indicate a lesion of the median nerve proximal to the branching point of the anterior interosseous nerve.

An Emergency Department physician examining a patient with a bullet wound in the arm, asks the patient to make a ring (O) sign with thumb and index finger tips touching each other. The physician noted ample flexion of the IP joint of the thumbb and the PIP and DIP joints of the index finger. This shows that which of the following nerves is intact? A Anterior interosseous B Posterior interosseous C Superficial Radial D Deep Radial E Superficial Ulnar F Deep Ulnar

B after providing motor innervation to the muscles of the anterior compartment of the arm, the musculocutaneous nerve terminates as the lateral cutaneous nerve of the forearm

An MSU undergrad is racing to class on his mountain bike, hits the chains at a railroad crossing, and is thrown over the handlebars, jerking his left arm before he lets go of the bike. The Emergency Department physician is concerned that there may be a traction injury to the brachial plexus. She decides to perform pin-prick tests to rule out damage to the nerve roots. All of the distal nerve roots assessed are without impairment. The patient describes numbness in the lateral forearm. The above findings indicate that which of the following nerves has been damaged? A. Axillary nerve B. Musculocutaneous nerve C. Radial nerve D. Median nerve E. Ulnar nerve

D: Intercostobrachial nerve. The intercosto- brachial nerve is the lateral cutaneous branch of the second intercostal nerve. As its name implies, it leaves the thorax by exiting between the second and third ribs (between the ribs = intercosto-) to supply cutaneous innervation to the axilla and medial aspect of the arm (brachium). In some instances, it may also supply skin distal to the elbow. The anesthetic solu- tion would block all of the distal branches of the brachial plexus residing within the axillary sheath, thus sparing the intercostobrachial nerve. Choice A (Long thoracic nerve) is incorrect. The long thoracic nerve does not have a cuta- neous distribution and provides only motor innervation to the serratus anterior. Because the long thoracic nerve arises from the ventral rami of C5-C7, it would not be affected by the anesthetic solution, especially if an occlusive tourniquet was utilized to retain the solution. Such a procedure is used to prevent spread of the anesthetic into the neck where it could affect the phrenic nerve and negatively affect respi- ration. Choice B (Median nerve) is incorrect. The median nerve does not branch proximal to the elbow, and its sensory distribution is limited to distal to the wrist. Furthermore, this nerve would be blocked by effective administration of the anesthetic reagent. Choice C (Medial cutaneous nerve of the arm) is incorrect. The medial cutaneous nerve of the arm arises from the medial cord of the brachial plexus, and it would be blocked by effective administration of the anes- thetic solution into the axillary sheath. This nerve would also provide sensation to the area (medial aspect of the arm) that remains sensitive to pain, but it should be blocked by the drug. Choice E (Ulnar nerve) is incorrect. The ulnar nerve does not branch proximal to the elbow, and its sensory dis- tribution is limited to distal to the wrist. Furthermore, this nerve would be blocked by effective administration of the anesthetic solution.

An anesthesiologist administers an anesthetic solution into the axillary sheath of a 19-year-old college baseball player in preparation for repair of the ulnar collateral ligament of the elbow. After 5 minutes, the patient experiences numbness and paresthesia distal to the middle aspect of the arm; however, the medial aspect of the arm and elbow remain sensitive to pain. What nerve provides sensory innervation to the sensitive area and was not blocked by the anesthetic solution? (A) Long thoracic nerve (B) Median nerve (C) Medial cutaneous nerve of the arm (D) Intercostobrachial nerve (E) Ulnar nerve

A Supraspinatus. The greater tubercle of the humerus is the insertion site of three (of the four) rotator cuff muscles: supraspinatus, infraspinatus, and teres minor. Avulsion of this structure could result in detachment of any of these rotator cuff muscles, depending upon the size and scope of the fracture. However, the wrestler is unable to initi- ate abduction of the upper limb, which implies damage to the supraspinatus muscle. Choice B (Long head of biceps brachii) is incorrect. This muscle originates from the supraglenoid tubercle of the scapula and passes between the greater and lesser tubercles of the humerus, in the intertubercular (bicipi- tal) groove. Detachment of the tendon of this muscle causes the biceps brachii to bulge in the anterior arm. Avulsion of the biceps brachii muscle is not related to the greater tubercle of the humerus. Choice C (Long head of triceps) is incor- rect. The long head of the triceps brachii muscle originates from the infraglenoid tubercle of the scapula and inserts on the olecranon process of the ulna. It would not be involvedin avulsion of the greater tubercle of the humerus. Choice D (Subscapularis) is incorrect. The fourth rotator cuff muscle, subscapularis, inserts onto the lesser tubercle of the humerus, so it would not be directly involved with this avulsion injury. Choice E (Infraspinatus) is incorrect. The infraspinatus mus- cle does insert onto the middle aspect of the greater tubercle of the humerus; however, damage to this muscle would result in weakness in external rotation at the shoulder joint, not the problems with abduction seen in this patient.

As a result of chronic stress associated with an intense high school weight-lifting program, a 15-year-old boy suffers an avulsion fracture of the greater tubercle of the humerus. In the ER, he displays difficulty initiating abduction of the upper limb. Which of the following muscles was involved in this fracture? (A) Supraspinatus (B) Long head of biceps brachii (C) Long head of triceps (D) Subscapularis (E) Infraspinatus

C: Brachialis muscle. The cubital fossa is a triangular intermuscular space located anterior to the elbow, comparable to the popliteal fossa in the lower limb. It is an important transition zone between the arm and the fore- arm, containing major blood vessels and nerves. The roof of the cubital fossa is subcutaneous tissue carrying superficial veins, such as the median cubital vein, which is the most common site for venipuncture in the upper limb. The floor of the fossa is formed by two muscles: brachialis and supina- tor. Either of these muscles could house the deep abscess caused by repeated needle injections in the cubital fossa. Choice A (Brachioradialis muscle) is incorrect. The brachi- oradialis muscle forms the lateral wall of the cubital fossa. Choice B (Pronator teres muscle) is incorrect. The prona- tor teres muscle forms the medial wall of the cubital fossa. Choice D (Head of the radius) is incorrect. The head of the radius lies deep to the muscular floor of the cubital fossa, within the elbow joint complex. Choice E (Olecranon fossa of the humerus) is incorrect. The olecranon fossa is located on the posterior distal aspect of the humerus, well removed from the cubital fossa.

Because of repeated bad needle sticks, a heroin addict develops an infected abscess in the floor of the cubital fossa. Which of the following structures is the abscess most likely to invade first? (A) Brachioradialis muscle (B) Pronator teres muscle (C) Brachialis muscle (D) Head of the radius (E) Olecranon fossa of the humerus

E Inability to adduct the thumb. The location of the cut, superficial to the flexor retinaculum, indicates injury to the ulnar nerve. This nerve controls most of the intrinsic muscles of the hand, including the adductor pollicis. Because this muscle is the sole adductor of the thumb, the result will be inability to adduct that digit. Choice A (Weakness in prona- tion) is incorrect. Pronation is controlled by branches of the median nerve to the pronator teres and pronator quadratus muscles. These are located proximal to the site of the injury. Choice B (Inability to abduct the thumb) is incorrect. Abduc- tion of the thumb is governed by the radial nerve (to abductor pollicis longus) and the recurrent branch of the median nerve (to abductor pollicis brevis) at sites removed from the injury. Choice C (Weakness in flexion of the thumb) is incorrect. Flexion of the thumb is controlled by branches of the median nerve at locations proximal (to flexor pollicis longus) and distal (to flexor pollicis brevis) to the cut flexor retinaculum (transverse carpal ligament). However, the reported laceration was superficial to this ligament and proximal to the pisiform bone, so the median nerve would not have been damaged. Choice D (Weakness in opposition of the thumb) is incorrect. Opposition of the thumb is controlled by the recurrent branch of the median nerve (to opponens pollicis), distal to the injury site.

During an attempted suicide, a depressed young woman slashes her wrist with a straight razor. She cuts just proximal to the pisiform bone to the depth of the superficial aspect of the flexor retinaculum before passing out at the sight of her own blood. As a result of this wound, she may suffer a neuromuscular deficit that results in which of the following? (A) Weakness in pronation (B) Inability to abduct the thumb (C) Weakness in flexion of the thumb (D) Weakness in opposition of the thumb (E) Inability to adduct the thumb

E: Flexor carpi radialis. The flexor retinaculum (transverse carpal ligament) is a thickening of investing deep fascia on the anterior (ventral) aspect of the wrist that forms the roof of the carpal tunnel. All issues related to the carpal tunnel revolve around an understanding of what structures are contained within the carpal tunnel versus the positions of structures outside the tunnel. The tendons of the flexor carpi radialis and palmaris longus muscles, plus the ulnar nerve and ulnar artery, lie against the superficial aspect of the flexor retinaculum. Any of these structures may be lesioned by a superficial cut across the front (anterior side) of the wrist. Choice A (Flexor digitorum superficialis) is incorrect.The tendons of this muscle are contained within the carpal tun- nel, deep to the flexor retinaculum. Choice B (Brachioradialis) is incorrect. This muscle inserts onto the base of the styloid process of the radius. Thus, its tendon is proximal and lateral to the flexor retinaculum, and therefore unrelated to the car- pal tunnel. Choice C (Flexor pollicis longus) is incorrect. The tendons of this muscle are contained within the carpal tunnel, deep to the flexor retinaculum. Choice D (Abductor pollicis longus) is incorrect. This muscle is a member of the poste- rior compartment of the forearm. Its tendon loops out of that compartment to attach onto the lateral side of the base of the first metacarpal (the lateral base of the thumb). Thus, it is well removed from the flexor retinaculum and the carpal tunnel.

During an attempted suicide, a depressed young woman slashes the front of her wrist with a razor blade. However, she cuts only to the depth of the superficial aspect of the flexor retinaculum before passing out at the sight of her own blood. Which of the following muscle tendons may be severed? (A) Flexor digitorum superficialis (B) Brachioradialis (C) Flexor pollicis longus (D) Abductor pollicis longus (E) Flexor carpi radialis

B,C,F The inferior trunk is composed of fibers from the C8-T1 ventral primary rami. The medial cutaneous nerve of the arm, medial cutaneous nerve of the forearm, and ulnar nerves are all composed exclusively of C8 and T1 fibers. The median and radial nerves carry some C8 and T1 fibers and would be partially compromised.

If the inferior trunk is injured, which of the following nerves would be completed compromised? (select all that apply) A axillary B medial cutaneous nerve of arm C medial cutaneous nerve of forearm D median E radial F ulnar

The answer is E: Teres minor. Throwing motions are complex mechanical events that involve multiple muscles interacting in moment-to-moment changing ways, with rotation of the humerus being one important outcome. The four rotator cuff muscles (supraspinatus, infrapsinatus, teres minor, subscapu- laris), plus several other muscles that cross the glenohumeral joint, contribute significant forces to rotation of the arm. The teres minor and infraspinatus (i.e., half the rotator cuff group) lie completely across the posterior aspect of the glenohumeral joint and are primary lateral (external) rotators. These mus- cles are aided by the posterior fibers of the deltoid muscle. Choice A (Supraspinatus) is incorrect. The supraspinatus is one of the rotator cuff muscles. However, it does not produce rotation. It lies across the superior aspect of the glenohumeral joint and initiates abduction of the arm from the rest position. Choice B (Teres major) is incorrect. The teres major arises pos- teriorly from the inferior angle of the scapula and crosses the glenohumeral joint to its anteriorly located insertion into the medial lip of the intertubercular groove of the humerus. Thus, the teres major muscle acts to adduct and medially (internally) rotate the arm rather than laterally rotate. It is not a member of the rotator cuff muscles. Choice C (Latissimus dorsi) is incorrect. The latissimus dorsi muscle arises posteriorly from the spinous processes of the inferior six thoracic vertebrae and the thoracolumbar fascia and crosses the glenohumeral joint anteriorly to insert into the floor of the intertubercular sul- cus of the humerus. Thus, the latissimus dorsi muscle acts to extend, adduct, and medially rotate the humerus rather than laterally rotate. It is not a member of the rotator cuff muscles. Choice D (Subscapularis) is incorrect. The subscapularis mus- cle is part of the rotator cuff. In common with the teres major and latissimus dorsi muscles, it crosses the glenohumeral joint from posterior to anterior, thus producing medial rotation and adduction of the arm. It is not responsible for external (lateral) rotation of the arm.

Lateral rotation of the arm is an important mechanical component of "bringing the arm back" when preparing to throw an object. What muscle acts to produce lateral rotation of the arm? (A) Supraspinatus (B) Teres major (C) Latissimus dorsi (D) Subscapularis (E) Teres minor

A: Median nerve. The median nerve controls pronation through the actions of the pronator teres and prona- tor quadratus muscles in the anterior compartment of the forearm. It also controls much of flexion of the wrist and lat- eral digits via the actions of most of the other muscles in that compartment. Choice B (Ulnar nerve) is incorrect. The ulnar nerve controls 11⁄2 muscles in the anterior compartment of the forearm (flexor carpi ulnaris and the ulnar half of the flexor digitorum profundus) and most of the intrinsic muscles of the hand. However, neither of these 11⁄2 forearm muscles pro- duces pronation. Choice C (Superficial branch of the radial nerve) is incorrect. The superficial branch of the radial nerve is entirely cutaneous, carrying sensation from the dorsolateral part of the hand. So, cutting this nerve would not result in weakness in pronation of the forearm or flexion of the wrist. Choice D (Deep branch of the radial nerve) is incorrect. The deep branch of the radial nerve supplies the posterior com- partment of the forearm. It influences supination via motor control of the supinator muscle, but not pronation. Choice E (Musculocutaneous nerve) is incorrect. The musculocutane- ous nerve supplies the anterior compartment of the arm, and then continues distally via its termination as the lateral cutane- ous nerve of the forearm. It contributes significantly to control of supination by its innervation of the biceps brachii muscle, but has no effect on wrist flexion.

Physical examination of a 40-year-old man injured in an auto- mobile accident indicates that he has suffered nerve damage affecting his left upper limb. The patient exhibits significant weakness when pronating his left forearm and flexing his left wrist. What nerve is most likely damaged? (A) Median nerve (B) Ulnar nerve (C) Superficial branch of the radial nerve (D) Deep branch of the radial nerve (E) Musculocutaneous nerve

D: Axillary nerve. The quadrangular space of the shoulder is an important passageway allowing the poste- rior humeral circumflex vessels and their companion axillary nerve to pass from the axilla to the posterior aspect of the shoulder. The neurovascular bundle runs across the surgical neck of the humerus to enter the quadrangular space. The space itself (sometimes termed the lateral axillary hiatus) is formed by four structures: teres major, teres minor, long head of the triceps, surgical neck of the humerus. The vessels con- tribute to the collateral network around the shoulder. The axil- lary nerve supplies the teres minor and deltoid muscles and a cutaneous area on the superolateral aspect of the arm (i.e., the skin overlying the lower aspect of the deltoid muscle). Choice A (Musculocutaneous nerve) is incorrect. The mus- culocutaneous nerve is a terminal branch of the lateral cord of the brachial plexus within the axilla. It supplies the anterior compartment of the arm and is not related to the quadrangular space. Choice B (Lateral cord of the brachial plexus) is incor- rect. The lateral cord occupies a relatively lateral position in the axilla but is not related to the quadrangular space. The axillary nerve originates from the posterior cord of the brachial plexus. Choice C (Radial nerve) is incorrect. The radial and axillary nerves are the terminal branches of the posterior cord of the brachial plexus. The radial nerve runs through the radial (spi- ral) groove in the midshaft of the humerus to emerge through the lower triangular space, just below the quadrangular space, and enter the posterior compartment of the arm. Choice E (Medial cutaneous nerve of the arm) is incorrect. The medial cutaneous nerve of the arm is a branch of the medial cord of the brachial plexus. It is far removed from the quadrangular space.

Physical examination of a 45-year-old man who had been stabbed in the back of the shoulder shows a deep wound penetrating into the quadrangular space of the shoulder, causing bleeding from the severed blood vessels there. Which of the following neural structures is most likely damaged as well? (A) Musculocutaneous nerve (B) Lateral cord of the brachial plexus (C) Radial nerve (D) Axillary nerve (E) Medial cutaneous nerve of the arm

A The long head of biceps brachii passes through the glenohumeral joint cavity to reach its insertion on the supraglenoid tubercle of the scapula.

Steroid injections into the shoulder joint space will most help with inflammation of which of the following tendons? A The long head of the biceps brachii B The short head of the biceps brachii C The coracobrachialis D The long head of the triceps brachii E The pectoralis minor

A: Posterior circumflex humeral artery. The radiograph shows a fracture of the surgical neck of the humerus. The posterior humeral circumflex artery, accompa- nied by the axillary nerve, lies against the posterior aspect of the surgical neck as it passes into the quadrangular space of the shoulder. This fracture places both of these structures in immediate danger. Choice B (Brachial artery) is incorrect. The brachial artery travels down the midline of the arm, close to the median nerve. It is not immediately endangered by the fracture of the surgical neck. Choice C (Deep brachial, or pro- funda brachii, artery) is incorrect. The deep brachial artery wraps tightly around the midshaft of the humerus, in the radial (spiral) groove. It, along with the radial nerve with which it travels, would be endangered by a fracture through the radial groove of the humerus. Choice D (Subscapular artery) is incor- rect. The subscapular artery is the largest branch of the axil- lary artery. It descends along the axillary border of the scapula and is not in contact with the humerus. Choice E (Superior ulnar collateral artery) is incorrect. This artery is a branch of the brachial artery. It descends through the arm, moves into company with the ulnar nerve, and takes a close relation to the posterior aspect of the medial epicondyle of the humerus. This vessel would be endangered by a fracture of the medial epicondyle rather than a surgical neck fracture.

The given X-ray reveals a fracture of the proximal humerus, indicated by the black arrow. Given the location of the fracture, what artery is most likely damaged in this patient? (A) Posterior circumfl ex humeral artery (B) Brachial artery (C) Deep brachial (profunda brachii) artery (D) Subscapular artery (E) Superior ulnar collateral artery

D Profunda brachii artery. It is important to recognize where neurovascular structures have close posi- tional relations to each other and to underlying bony struc- tures in order to predict the likely second order functional consequences of damage to the bones. In the given AP X-ray, the midshaft of the humerus is fractured slightly distal to the radial groove. At this point, the profunda brachii vessels (deep vessels of the arm) and the radial nerve emerge from the radial groove in a bundle tightly wrapped against the body of the humerus. A fracture here may readily damage any of these neurovascular structures. Lesion of the vessels may produce swelling in the posterior compartment of the arm and loss of supply to the muscles therein. Lesion of the nerve will result in major motor and sensory deficits in the posterior aspect of the forearm and hand. Choice A (Posterior circumflex humeral artery) is incorrect. This vessel travels in companionship with the axillary nerve around the surgical neck of the humerus. Fractures here are the most common injuries to the proximal end of the humerus, especially in the elderly. Choice B (Ulnar nerve) is incorrect. In the arm, the ulnar nerve travels with the superior ulnar collateral artery. Both lie in contact with the posterior side of the large, projecting medial epicondyle of the humerus as they cross the elbow. Choice C (Axillary nerve) is incorrect. The axillary nerve travels in companionship with the posterior circumflex humeral artery around the surgical neck of the humerus. Fractures here are the most common injuries to the proximal end of the humerus, especially in the elderly. Choice E (Median nerve) is incorrect. This large nerve travels with the brachial vessels down the anterior midline of the arm. The nerve lies close to the distal end of the humerus, where it may be damaged due to fractures of the condyle.

The given anteroposterior (AP) X-ray depicts a humeral shaft fracture in a 22-year-old man. Given the location of the fracture, which of the following structures is most likely damaged? (A) Posterior circumfl ex humeral artery (B) Ulnar nerve (C) Axillary nerve (D) Profunda brachii artery (E) Median nerve

B: Axillary artery. The brachial plexus is divided into five geographic parts: Roots (or ventral rami of C5-T1), Trunks, Divisions, Cords, terminal Branches (or Nerves). These sections of the brachial plexus can be remem- bered with the mnemonics, "Real Truckers Drink Cold Beer" or "Remember Those Darn Cervical Nerves". The cords are named according to their important positional relationship to the second part of the axillary artery, deep to the pectoralis minor muscle. Here, the nerves form a cradle-like bed for this segment of the vessel as it passes through the axilla. Thus, the lateral cord is located lateral, the medial cord is medial, and the posterior cord is posterior to the axillary artery. Choice A (Long head of the biceps brachii muscle) is incor- rect. The long head of the biceps brachii muscle lies lateral to all three cords of the brachial plexus. Choice C (Subclavian vein) is incorrect. This vessel is located well proximal to the cords of the brachial plexus, medial to the first rib. Choice D (Surgical neck of the humerus) is incorrect. This part of the humerus is located distal and lateral to all three cords of the brachial plexus. Choice E (Pectoralis minor muscle) is incorrect. The pectoralis minor lies superficial (anterior) to the cords of the brachial plexus. This muscle divides the axillary artery into its three parts. Thus, it defines the second part of the artery and creates the situation for naming the cords of the plexus.

The lateral cord of the brachial plexus is named because it lies immediately lateral to which of the following structures? (A) Long head of the biceps brachii muscle (B) Axillary artery (C) Subclavian vein (D) Surgical neck of the humerus (E) Pectoralis minor muscle

B: Second part of the axillary artery. The lat- eral thoracic wall receives significant arterial supply from the branches of the axillary artery. The axillary artery is divided into three parts by the overlying pectoralis minor muscle. The lateral thoracic artery, along with the thoracoacromial trunk, typically branches from the second part of the axillary artery, deep to the pectoralis minor. It descends along the lateral border of the pectoralis minor to supply the lateral aspect of the chest wall, including much of the breast and the serra- tus anterior muscle. However, this vessel is variable and may originate from other source points. Remember, arteries are often named for the regions they supply, not necessarily for their branching patterns. Choice A (First part of the axillary artery) is incorrect. This segment typically gives rise to one branch: the superior (or supreme) thoracic artery, which has a small distribution to the superolateral chest wall. Choice C (Third part of the axillary artery) is incorrect. This part typi- cally gives rise to three branches: the subscapular artery and the anterior and posterior circumflex humeral arteries. The lateral thoracic and subscapular arteries may originate as a common trunk. Choice D (Third part of the subclavian artery) is incorrect. The subclavian artery is divided into three parts by the anterior scalene muscle. Usually, the third part of the subclavian artery has no branches; however, the dor- sal scapular artery may occasionally arise from this location. More importantly, none of the branches of the subclavian artery supply the lateral chest wall. Choice E (First part of the brachial artery) is incorrect. The distribution of the bra- chial artery is limited to the upper limb; it does not supply the lateral chest wall.

The lateral thoracic artery provides the main blood supply to the lateral side of the chest wall, including much of the breast. To deter excessive blood loss during a surgical procedure involving the breast, a surgeon can clamp the lateral thoracic artery near its origin. Which of the following arteries gives rise to this artery? (A) First part of the axillary artery (B) Second part of the axillary artery (C) Third part of the axillary artery (D) Third part of the subclavian artery (E) First part of the brachial artery

A,B,C,F

Which of the following muscles are responsible for flexion at the glenohumeral joint? (select all that apply) A biceps brachii long head B coracobrachialis C deltoid D infraspinatus E latissimus dorsi F pectoralis major G subscapularis H supraspinatus I teres major J teres minor

C,D,J

Which of the following muscles are responsible for lateral rotation of the humerus? (select all that apply) A biceps brachii long head B coracobrachialis C deltoid D infraspinatus E latissimus dorsi F pectoralis major G subscapularis H supraspinatus I teres major J teres minor

C,E,F,G,I

Which of the following muscles are responsible for medial rotation of the humerus? (select all that apply) A biceps brachii long head B coracobrachialis C deltoid D infraspinatus E latissimus dorsi F pectoralis major G subscapularis H supraspinatus I teres major J teres minor

C, F,H

Which of the following muscles of the forearm are innervated by the anterior interosseous nerve? A Flexor carpi radialis B Flexor carpi ulnaris C Flexor digitorum profundus lateral half D Flexor digitorum profundus medial half E Flexor digitorum superficialis F Flexor pollicis longus G Pronator teres H Pronator quadratus

B,D Both the median and ulnar nerves innervate muscles of the anterior compartment of the forearm and the palm of the hand. Most of the muscles of the anterior forearm are innervated by the median nerve; most of the muscles of the palm of the hand are innervated by the ulnar nerve.

Which of the following muscles of the forearm are innervated by the ulnar nerve? A Flexor carpi radialis B Flexor carpi ulnaris C Flexor digitorum profundus lateral half D Flexor digitorum profundus medial half E Flexor digitorum superficialis F Flexor pollicis longus G Pronator teres H Pronator quadratus


Ensembles d'études connexes

Prin of Entrepreneurship Exam 2 - MindTap Questions

View Set

Expert (Video Case 11) Witness: Starbucks

View Set

MedSurg Chapter 36: Introduction to the Nervous System

View Set

BUS312: Ch.5: Consumer Markets and Buyer Behavior (REVIEW)

View Set

Major components of gastric juice

View Set

Science Chapter 14+15 / Matching

View Set